Past LSAT Explained/PrepTest 43
October 2001 Form 1LSS51
Section I Reading Comprehension
[edit | edit source]Passage 1: Science Passage: Oil Well Drilling
Question 01
[edit | edit source]This is a Main Idea question.
(A) Incorrect. "...without appropriate safeguards, wells that penetrate both groundwater and oil or saline water formations inevitably contaminate the groundwater." (Lines 30-33) This implies that there exist safe guards, and in fact, the passage goes on to describe a safeguard that protects and groundwater contamination--hence, the contamination is not "inevitable whenever drilling for oil occurs".
(B) Incorrect. This is a restatement of the last sentence of the passage, and this topic is not addressed in any of the other paragraphs.
(C) Incorrect. The passage does not state the hindsight has been the only reliable way, it says "most attempts to regulate the industry have relied on hindsight."
(D) Correct. This correctly summarizes the main point of the passage. This can be seen by examining the first paragraph.
(E) Incorrect. The passage states that this used to be the case, but no longer is.
The first paragraph of the passage can often, especially in a scientific article, help key one into the main point of the passage (Consider the five paragraph essay format).
Question 02
[edit | edit source](A) Incorrect. There is no evidence provided in the passage for this statement.
(B) Incorrect. There is no evidence provided in the passage for this statement; in fact, the passage discusses several oil wells in or near coastal regions.
(C) Incorrect. There is no evidence provided in the passage for this statement.
(D) Incorrect. "...regulations reflected the industry's concern for the purity of the well's oil" (12-14). This does not necessarily mean that oil wells are usually contaminated.
(E) Correct. "...groundwater is usually found...near the earth's surface, whereas petroleum...[is] generally found in similar formations but at greater depths" (24-28).j
Question 03
[edit | edit source]Author's attitude
(A) This response is too restricting by limiting it to "without international concern." The last paragraph gives example of undesirable events that will/did lead to more research, but these are not restricted to events causing "international concern".
(B) There is no evidence in the passage to suggest the author thinks this, in fact, the last paragraph clearly shows the authors concern that more undesirable events will occur.
(C) Correct. This is best gleaned from the tone and examples from the last paragraph.
(D) Incorrect. "...there is no guarantee that wells drilled in compliance with existing regulations will not expose a need for research in additional areas" (52-54). The author seems to think the opposite, that an event will show that regulation is inadequate and spur research.
(E) Incorrect. "...there is no guarantee that wells drilled in compliance with existing regulations will not expose a need for research in additional areas" (52-54).
Question 04
[edit | edit source]Question 05
[edit | edit source]Question 06
[edit | edit source]Question 07
[edit | edit source]Question 08
[edit | edit source]Question 09
[edit | edit source]Question 10
[edit | edit source]Question 11
[edit | edit source]Question 12
[edit | edit source]Question 13
[edit | edit source]Question 14
[edit | edit source]Question 15
[edit | edit source]Question 16
[edit | edit source]Question 17
[edit | edit source]Question 18
[edit | edit source]Question 19
[edit | edit source]Question 20
[edit | edit source]Question 21
[edit | edit source]Question 22
[edit | edit source]Question 23
[edit | edit source]This question is asking what you can infer from the passage about the author's viewpoint.
(A) This is the credited response. Line 56-58 state that "most major institutions behave in the ways that maximize university ownership and profit participation."
(B) Lines 4-7 point out that "many institutions have invested heavily in the...infrastructure to develop...these discoveries", implying that their mission is in line with making the discoveries, not antithetical to it (against it).
(C) The purpose of the quote in lines 16-19 is to show that if universities don't consider their policies, they could lose their professors to institutions "responsive to their commercialized desires". This implies that the policies do have an impact on the research of the faculty.
(D) Just because the policies generally work to the benefit of the universities doesn't mean they will definitely harm the professors, just that they will tend to benefit the universities more than the professors. A counter-example could be a resource-provider university (line 40), where it certainly seems to be more of a gray area.
(E) Line 52 points out that this is an area of uncertain legal boundaries, so it may or may not be illegal - we don't know.
Question 24
[edit | edit source]Question 25
[edit | edit source]Question 26
[edit | edit source]Question 27
[edit | edit source]Question 28
[edit | edit source]Section II Logical Reasoning
[edit | edit source]Question 01
[edit | edit source]Question 02
[edit | edit source]Question 03
[edit | edit source]Question 04
[edit | edit source]Question 05
[edit | edit source]Question 06
[edit | edit source]Question 07
[edit | edit source]This question is a must be true inference question. In other words, the answer must be an inference that must be true in all possible situations. The stimulus gives a number of details regarding crops and vegetables. There is no conclusion here, just information. This question is made difficult by the length of the stimulus and the confusing conditional statements. We learn the following details from the stimulus:
• Unless preventative measures (left unspecified) are taken, failure to rotate crops will lead to a depletion of soil nutrients.
• If the soil's nutrients are completely depleted, fertilizer is necessary to grow crops.
• If vegetables are grown in fertilized soil instead of non-fertilized soil, they are more vulnerable to pests and must be treated with more pesticides.
• The more pesticides used on veggies, the greater health risks to humans eating the veggies.
The prompt then asks what must be true for some vegetables that were grown in unfertilized soil.
(A): This answer is incorrect because the scenario given in this answer could never be true. The second conditional statement says that fertilizer is necessary to grow crops in the case of nutrition depletion in the soil. The prompt states that the soil is not fertilized. Therefore, if the soil was depleted of nutrients, no crops (including vegetables) could have grown.
(B): If the crops have been rotated, then we know that there will not be a depletion of soil nutrients due to a failure to rotate crops. This leaves open the possibility of the soil nutrients remaining. This possibility does not conflict with any of the other conditions given in the stimulus or prompt. Therefore, (B) is incorrect. There is nothing in the stimulus that makes this scenario impossible.
(C): This is guaranteed by the conditions given in the stimulus. The first condition states that if the crop rotation failure will lead to soil nutrition depletion "'unless preventative measures are taken.'" In other words, a failure to rotate crops need not inevitably lead to the nutrients being depleted from the soil. We know that there are nutrients in the soil because the vegetables are being grown without fertilizer. If the nutrients were completely depleted, the second conditional statement would be violated. The fact that there are nutrients in the soil, however, does not tell us for sure whether or not someone rotated the crops. It may be that someone failed to rotate the crops but performed preventative maintenance. The given information is not enough to know if the crops were rotated. Therefore, the possibility invariably exists that the crops weren't rotated but the soil still has nutrition. Therefore, (C) must be true.
Identifying the correct answer here is made a little more difficult by the fact that choosing (C) as the correct answer is basically saying, "The possibility that the vegetables were grown in soil of crops not rotated is guaranteed to be true." Guaranteeing the possibility of something is a lot less directing than simply guaranteeing that something is true.
(D): There are two main ways in which this answer is incorrect. Although the third condition of the stimulus guarantees that more pesticides would be used on vegetables grown in fertilized soil, there is nothing stating that no pesticides would be used on vegetables in non-fertilized soil. It could be that a ton of pesticides are already used on veggies in non-fertilized soil, and just a little bit more than a ton is used on veggies in fertilized soil. "More" is an ambiguous term because it is relative. It tells us nothing about the absolute values. Furthermore, we don't know the amount of pesticide applied to vegetables in non-fertilized soil is below the threshold of toxicity to humans. In other words, it could be that even a tiny amount of pesticide use is toxic to humans, and that toxicity goes up as more pesticide is used (as stated in the fourth condition). Since the scenario given in the prompt leaves open the possibility that any amount of pesticide could be used on the vegetables grown in non-fertilized soil and that the tiniest amount of pesticide is a health risk to humans, (D) may or may not be true. Thus, (D) is incorrect. (Remember, the correct answer must be guaranteed to be true)
(E): Again, we have no idea what amount of pesticides will be applied to these vegetables in non-fertilized soil. Thus, we can't make any kind of comparisons with other vegetables. (E) may or may not be true is thus, incorrect.
Question 08
[edit | edit source]Question 09
[edit | edit source]Question 10
[edit | edit source]Question 11
[edit | edit source]Question 12
[edit | edit source]Question 13
[edit | edit source]Question 14
[edit | edit source]This is a strengthen question. That is, what information, if added, would support the given conclusion. The conclusion of the argument is that "residents of this locale should not consider their loss of farming as a way of life to be a tragedy." The evidence is that it was economically depressed when it was a rural area, but now it is a high tech area that supports many jobs.
(A) This answer is outside the scope of the argument, ie, it is irrelevant to the conclusion.
(B) Again, this is irrelevant. We don't know or care about national security.
(C) This is the credited response. This is correct because it fills in a major gap left by the argument, that is, regardless of their newfound prosperity, do they have any other reason to miss farming as a way of life? Since this statement is that they don't (inherently) value farming any more than economic prosperity, it strengthens the argument.
(D) This is the sucker punch - many readers will assume that this is an improvement, and therefore strengthens the argument. Although it is more evidence along the same lines as that already given in the passage, it doesn't help to directly support the conclusion that "residents should not consider the loss of farming as a...tragedy"
(E) This would actually weaken the argument. Even though no financial hardship is resulting, if this were true, the farmers would definitely still consider the loss of their way of life a tragedy.
Question 15
[edit | edit source]This is a resolve question. There is an apparent contradiction between the two statements, so you are looking for an additional piece of evidence that makes the other two statements work together.
(A) This is the opposite of what both statements are arguing - they are saying cigarette ads shouldn't be illegal because fatty foods ads aren't illegal, not that both should be illegal.
(B) This doesnt resolve the contradiction, it merely offers additional support for cigarette ads being blocked and fatty foods not being blocked, which doesnt help because fatty foods are still unhealthy.
(C) The first statement implies the government should have some kind of moral responsibility, not the advertisers, so this doesnt resolve anything.
(D) This is the credited response. This is what would really resolve the idea. The two statements are saying that the government is justified in trying to limit the numbers of ads, but it making them illegal would be going too far. The solution of providing financial disincentives would fulfill the government's obligation to try to prevent ads that encourage people from engaging in an unhealthy practice (statement 1), but still not make it illegal because there are other unhealthy practices that can be legally advertised (statement 2).
(E) This doesn't resolve the contradiction in the idea that if government is trying to keep ads from encouraging people to participate in unhealthy practices, then they should discourage cigarette ads as well as fatty foods ads. It is correct in that it is saying they should restrict but not prevent, another element of the two arguments, but it isnt quite there.
Question 16
[edit | edit source]This is a necessary assumption question. The correct answer will be a piece of evidence that is required for the claim to stand. Without the correct answer, the argument falls apart.
In the stimulus, an environmentalist argues that "government mandated environmental protection can help those regions' economies overall, even if such protection harms some older local industries." This claim is based off of the following premises:
• Many people prefer living in regions of natural beauty.
• These regions often have an influx of new residents.
• Population growth encourages businesses to relocate to these regions.
The environmentalist has made quite a jump to make claims about helping the economies overall, even if the older local industries are harmed. The correct answer will probably address these new elements.
(A): Adding this piece of evidence to the argument would destroy the argument. If (A) were indeed true, then government mandated protection could not help the economies overall because new businesses would stop relocating to the region if older local industries were harmed. The environmentalist has claimed that the economy would be helped notwithstanding the harming of local industries. Therefore, (A) is incorrect.
"'(B)'": Even if the economies of these regions were based primarily on local industries that would be harmed by the government mandates, the argument can still stand as long as the new incoming businesses buoy up the economy enough to make up for the damages to the local industries. If there weren't enough new incoming businesses, (B) might be a necessary assumption but that's not good enough to make (B) the correct answer. A necessary assumption is an assumption that is guaranteed in all cases to be required. (B) is incorrect.
"'(C)'": The environmentalist's argument never required encouraging an influx of new businesses to be the primary means for a govt mandate to help a region's economy. A government mandate for environmental protection could help a region's economy in many other ways. The environmentalist has only addressed one potential outcome of such a mandate. There are many other possible outcomes that the environmentalist never denied the existence of. (C) is not required to be true and is thus, incorrect.
"'(D)'": The environmentalist never stated that a government mandated protection of the environment was the most beneficial to a region's economy. Other means could be more beneficial and the argument could still stand. Therefore, (D) is not a required assumption and is incorrect.
"'(E)'": If it was true that a factor harmful to some older local industries in a region "discouraged" other businesses from relocating to the region, then the argument would not stand. The environmentalist has made an assumption that a potential side effect of the government mandate will not prevent the desired boost to the economy. If the environmentalist is wrong, then the given premises can no longer support the claim. (E) is required and is the correct answer.
Question 17
[edit | edit source]Question 18
[edit | edit source]Question 19
[edit | edit source]Question 20
[edit | edit source]Question 21
[edit | edit source]Question 22
[edit | edit source]Question 23
[edit | edit source]Question 24
[edit | edit source]Question 25
[edit | edit source]Section III Logical Reasoning
[edit | edit source]Question 01
[edit | edit source]Question 02
[edit | edit source]Question 03
[edit | edit source]Question 04
[edit | edit source]Question 05
[edit | edit source]Question 06
[edit | edit source]Question 07
[edit | edit source]Question 08
[edit | edit source]Question 09
[edit | edit source]Question 10
[edit | edit source]Question 11
[edit | edit source]Question 12
[edit | edit source]Question 13
[edit | edit source]Question 14
[edit | edit source]Question 15
[edit | edit source]Question 16
[edit | edit source]Question 17
[edit | edit source]Question 18
[edit | edit source]Question 19
[edit | edit source]Question 20
[edit | edit source]Question 21
[edit | edit source]Question 22
[edit | edit source]Question 23
[edit | edit source]Question 24
[edit | edit source]Question 25
[edit | edit source]This is a parallel reasoning question. The reasoning in the given argument is one of mutual exclusivity. If good things cause no harm at all, and wealth is often harmful to people, then it is impossible for wealth to ever be a good thing. So we have to find an answer that uses that same line of reasoning.
(A) This is the credited response. Alex loves to golf, and no one in the chess club loves to golf, so it is impossible for Alex to be in the chess club, since he loves to golf.
(B) There is no exclusivity here; Isabella can be a contented, smiling, hardly ever crying baby, and therefore be like happy people.
(C) This doesn't state what would be mutually exclusive. LSAC wants us to assume that industry's growth would cause unbearable levels of pollution, and they want us to assume that would be a bad thing for the town. But they don't say either of those things, so as far as we know, none of those things are true, and there is nothing mutually exclusive here.
(D) This one matches on the structure of the sentence, but its reasoning is not the same. Even if most daschunds hunt poorly, that still leaves room for some that could hunt well (like Sarah's).
(E) Even though there is usually more traffic at this time of day, there can still be little traffic. Who cares if it is surprising or not: there is nothing that makes it impossible to happen.